Rozwiązać układ równań (5 niewiadomych)

Przestrzenie wektorowe, bazy, liniowa niezależność, macierze.... Formy kwadratowe, twierdzenia o klasyfikacji...
y=f(x)
Użytkownik
Użytkownik
Posty: 14
Rejestracja: 25 kwie 2009, o 20:12
Płeć: Mężczyzna
Lokalizacja: Gdańsk
Podziękował: 1 raz

Rozwiązać układ równań (5 niewiadomych)

Post autor: y=f(x) »

Sprawa życia i śmierci, bardzo proszę o rozwiązanie lub chociaż udzielenie przejrzystych wskazówek. Z góry dziękuję za poświęcony czas. Pozdrawiam,

\(\displaystyle{ \begin{cases} 2x - y + 3z + s - t = 1 \\ x + z + t = 0 \\ 2y - 3s + t = 3 \\ x + 2y - 4t = 2 \\ 3x - 2y + z - s + 3t = 0 \\ 2s + 3t = 1 \end{cases}}\)
agulka1987
Użytkownik
Użytkownik
Posty: 3090
Rejestracja: 24 paź 2008, o 15:23
Płeć: Kobieta
Lokalizacja: Opole
Podziękował: 1 raz
Pomógł: 879 razy

Rozwiązać układ równań (5 niewiadomych)

Post autor: agulka1987 »

\(\displaystyle{ \begin{bmatrix}2&-1&3&1&-1 \left|1\\1&0&1&0&1 \left|0\\0&2&0&-3&1 \left|3\\1&2&0&0&-4\left|2\\3&-2&1&-1&3\left|0\\0&0&0&2&3\left|1\end{bmatrix}}\)

zamiana \(\displaystyle{ w_{1} \ z \ w_{2} = \begin{bmatrix}1&0&1&0&1 \left|0\\2&-1&3&1&-1 \left|1\\0&2&0&-3&1 \left|3\\1&2&0&0&-4\left|2\\3&-2&1&-1&3\left|0\\0&0&0&2&3\left|1\end{bmatrix}}\)}

\(\displaystyle{ w_{2}-2w_{1}, w_{4}-w_{1}, w_{5}-3w_{1} = \begin{bmatrix}1&0&1&0&1 \left|0\\0&-1&1&1&-3 \left|1\\0&2&0&-3&1 \left|3\\0&2&-1&0&-5\left|2\\0&-2&-2&-1&0\left|0\\0&0&0&2&3\left|1\end{bmatrix}}\)

\(\displaystyle{ w_{2} \cdot (-1) = \begin{bmatrix}1&0&1&0&1 \left|0\\0&1&-1&-1&3 \left|-1\\0&2&0&-3&1 \left|3\\0&2&-1&0&-5\left|2\\0&-2&-2&-1&0\left|0\\0&0&0&2&3\left|1\end{bmatrix}}\)

\(\displaystyle{ w_{3}-2w_{2}, w_{4}-2w_{2},w_{5}+2w_{2} = \begin{bmatrix}1&0&1&0&1 \left|0\\0&1&-1&-1&3 \left|-1\\0&0&-2&-1&-5 \left|5\\0&0&1&-2&-11\left|4\\0&0&-4&-3&6\left|-2\\0&0&0&2&3\left|1\end{bmatrix}}\)

zamiana \(\displaystyle{ w_{3} \ z \ w_{4} = \begin{bmatrix}1&0&1&0&1 \left|0\\0&1&-1&-1&3 \left|-1\\0&0&1&-2&-11\left|4\\0&0&-2&-1&-5 \left|5\\0&0&-4&-3&6\left|-2\\0&0&0&2&3\left|1\end{bmatrix}}\)

\(\displaystyle{ w_{1}-w_{3}, w_{2}+w_{3}, w_{4}+2w_{3}, w_{5}+4w_{3} = \begin{bmatrix}1&0&0&2&2 \left|-4\\0&1&0&-3&-7\left|3\\0&0&1&-2&-11\left|4\\0&0&0&-5&-27 \left|13\\0&0&0&-11&-38\left|14\\0&0&0&2&3\left|1\end{bmatrix}}\)

jezeli nie pomyliłam się przy obliczeniach to układ ten jest sprzeczny
Ostatnio zmieniony 7 kwie 2010, o 21:59 przez agulka1987, łącznie zmieniany 1 raz.
Awatar użytkownika
scyth
Użytkownik
Użytkownik
Posty: 6392
Rejestracja: 23 lip 2007, o 15:26
Płeć: Mężczyzna
Lokalizacja: Warszawa
Podziękował: 3 razy
Pomógł: 1087 razy

Rozwiązać układ równań (5 niewiadomych)

Post autor: scyth »

\(\displaystyle{ x + z + t = 0 \quad \Rightarrow \quad z=-x-t \\
\begin{cases}
- x - y + s - 4t = 1 \\
2y - 3s + t = 3 \\
x + 2y - 4t = 2 \\
2x - 2y - s + 2t = 0 \\
2s + 3t = 1
\end{cases} \\
x + 2y - 4t = 2 \quad \Rightarrow x=2-2y+4t \\
\begin{cases}
y + s - 8t = 3 \\
2y - 3s + t = 3 \\
- 6y - s + 10t = -4 \\
2s + 3t = 1
\end{cases} \\
y + s - 8t = 3 \quad \Rightarrow y=3+8t-s \\
\begin{cases}
- 5s + 17t = -3 \\
5s - 38t = 14 \\
2s + 3t = 1
\end{cases}}\)

A ostatni układ jest sprzeczny, więc podany układ równań również jest sprzeczny.
y=f(x)
Użytkownik
Użytkownik
Posty: 14
Rejestracja: 25 kwie 2009, o 20:12
Płeć: Mężczyzna
Lokalizacja: Gdańsk
Podziękował: 1 raz

Rozwiązać układ równań (5 niewiadomych)

Post autor: y=f(x) »

Dlaczego układ ten jest sprzeczny? Wszak (w metodzie Agulki) w ostatnim wierszu nie występują same zera...
Awatar użytkownika
scyth
Użytkownik
Użytkownik
Posty: 6392
Rejestracja: 23 lip 2007, o 15:26
Płeć: Mężczyzna
Lokalizacja: Warszawa
Podziękował: 3 razy
Pomógł: 1087 razy

Rozwiązać układ równań (5 niewiadomych)

Post autor: scyth »

Ale łatwo możesz do tego sam doprowadzić.
agulka1987
Użytkownik
Użytkownik
Posty: 3090
Rejestracja: 24 paź 2008, o 15:23
Płeć: Kobieta
Lokalizacja: Opole
Podziękował: 1 raz
Pomógł: 879 razy

Rozwiązać układ równań (5 niewiadomych)

Post autor: agulka1987 »

\(\displaystyle{ \begin{bmatrix}2&-1&3&1&-1 \left|1\\1&0&1&0&1 \left|0\\0&2&0&-3&1 \left|3\\1&2&0&0&-4\left|2\\3&-2&1&-1&3\left|0\\0&0&0&2&3\left|1\end{bmatrix}}\)

zamiana \(\displaystyle{ w_{1} \ z \ w_{2} = \begin{bmatrix}1&0&1&0&1 \left|0\\2&-1&3&1&-1 \left|1\\0&2&0&-3&1 \left|3\\1&2&0&0&-4\left|2\\3&-2&1&-1&3\left|0\\0&0&0&2&3\left|1\end{bmatrix}}\)

\(\displaystyle{ w_{2}-2w_{1}, w_{4}-w_{1}, w_{5}-3w_{1} = \begin{bmatrix}1&0&1&0&1 \left|0\\0&-1&1&1&-3 \left|1\\0&2&0&-3&1 \left|3\\0&2&-1&0&-5\left|2\\0&-2&-2&-1&0\left|0\\0&0&0&2&3\left|1\end{bmatrix}}\)

\(\displaystyle{ w_{3}+2w_{2}, w_{4}+2w_{2}, w_{5}-2w_{2} = \begin{bmatrix}1&0&1&0&1 \left|0\\0&-1&1&1&-3 \left|1\\0&0&2&-1&-5 \left|5\\0&0&1&2&-11\left|4\\0&0&-4&-3&6\left|-2\\0&0&0&2&3\left|1\end{bmatrix}}\)

\(\displaystyle{ w_{2} \cdot (-1), \ zamiana \ w_{3} \ z \ w_{4} = \begin{bmatrix}1&0&1&0&1 \left|0\\0&1&-1&-1&3 \left|-1\\0&0&1&2&-11\left|4\\0&0&2&-1&-5 \left|5\\0&0&-4&-3&6\left|-2\\0&0&0&2&3\left|1\end{bmatrix}}\)

\(\displaystyle{ w_{1}-w_{3}, w_{2}+w_{3}, w_{4}-2w_{3}, w_{5}+4w_{3} = \begin{bmatrix}1&0&0&-2&12 \left|-4\\0&1&0&1&-8\left|3\\0&0&1&2&-11\left|4\\0&0&0&-5&17 \left|-3\\0&0&0&5&-38\left|14\\0&0&0&2&3\left|1\end{bmatrix}}\)

\(\displaystyle{ w_{4} \cdot \left( - \frac{1}{5} \right)= \begin{bmatrix}1&0&0&-2&12 \left|-4\\0&1&0&1&-8\left|3\\0&0&1&2&-11\left|4\\0&0&0&1&- \frac{17}{5} \left| \frac{3}{5} \\0&0&0&5&-38\left|14\\0&0&0&2&3\left|1\end{bmatrix}}\)

\(\displaystyle{ w_{1}+2w_{4}, w_{2}-w_{4},w_{3}-2w_{4}, w_{5}-5w_{4}, w_{6}-2w_{4}=\begin{bmatrix}1&0&0&0& \frac{26}{5} \left|- \frac{14}{5} \\0&1&0&0&- \frac{23}{5} \left| \frac{12}{5} \\0&0&1&0&- \frac{21}{5} \left| \frac{14}{5} \\0&0&0&1&- \frac{17}{5} \left| \frac{3}{5} \\0&0&0&0&-29\left|11\\0&0&0&0& \frac{49}{5} \left|- \frac{1}{5} \end{bmatrix}}\)

\(\displaystyle{ w_{5} \cdot \left(- \frac{1}{29} \right), w_{6} \cdot \left( \frac{5}{49} \right)=\begin{bmatrix}1&0&0&0& \frac{26}{5} \left|- \frac{14}{5} \\0&1&0&0&- \frac{23}{5} \left| \frac{12}{5} \\0&0&1&0&- \frac{21}{5} \left| \frac{14}{5} \\0&0&0&1&- \frac{17}{5} \left| \frac{3}{5} \\0&0&0&0&1\left|- \frac{11}{29} \\0&0&0&0& 1 \left|- \frac{1}{49} \end{bmatrix}}\)


jak widać wiersza 6 nie da się wyzerować więc układ jest sprzeczny
cmroz
Użytkownik
Użytkownik
Posty: 1
Rejestracja: 16 kwie 2010, o 23:01
Płeć: Mężczyzna

Rozwiązać układ równań (5 niewiadomych)

Post autor: cmroz »

Serdecznie pozdrawiam Inżynierię Środowiska,
C. M.
ODPOWIEDZ